Show that $forall n inBbb N: e < left(1+{1over n}right)^n left(1 + {1over 2n}right)$












11












$begingroup$



Show that:
$$
forall n inBbb N: e < left(1+{1over n}right)^n left(1 + {1over 2n}right)
$$




Till now i've only worked out a couple of proof sketches, and I don't have an idea how to proceed with them.



Please note that this question has already been asked here. The answer there uses derivatives and integrals which i'm not allowed to use.



First sketch



Consider the sequence:
$$
x_n = left(1+{1over n}right)^n left(1 + {1over 2n}right)
$$



One of the ways to show what's required is to show that:
$$
x_{n+1} le x_n
$$

namely the sequence is monotonically decreasing. Now given $ninBbb N$ we may calculate $x_1$:
$$
x_1 = left(1+{1over 1}right)^1left(1+{1over 2cdot 1}right) = 3
$$



Consider the limit:
$$
lim_{ntoinfty}x_n = lim_{ntoinfty} left(1+{1over n}right)^n left(1 + {1over 2n}right) = e
$$



Now based on the fact that the sequence tends to $e$ and it is monotonically decreasing and $x_1 = 3$, then it should follow that:
$$
forall ninBbb N: x_n ge e
$$

Here comes the hard part, I could't prove that $x_n$ is monotonically decreasing. I've considered the fraction:
$$
{x_{n+1}over x_n} = left(1 - {1over (n+1)^2}right)^n cdot frac{2n(2n+3)(n+2)}{(2n+2)(2n+1)(n+1)}
$$



Not sure how to show it is less than $1$.



Second sketch



This sketch is based on the idea from my previous question. Namely it has been shown there that:
$$
e le frac{n+2}{(n+1)(n+1)!} + sum_{k=0}^n {1over k!}
$$



It looks like:
$$
e le frac{n+2}{(n+1)(n+1)!} + sum_{k=0}^n {1over k!} le left(1+{1over n}right)^n left(1 + {1over 2n}right) tag1
$$



In such case if we prove $(1)$ we are done. I've given it several tries but it gets ugly very soon.



The question is:




Is it possible to utilize any of those sketches to finish the proof? If not what would be the way to prove the inequality using anything before the definition of a derivative/Taylor series/intergrals




Thank you!










share|cite|improve this question









$endgroup$












  • $begingroup$
    why not use the Principle of Mathematical Induction? It's just 4 lines if you use that :P XD :)
    $endgroup$
    – Abhas Kumar Sinha
    Jan 25 at 15:27










  • $begingroup$
    @AbhasKumarSinha I would appreciate if you could share that idea
    $endgroup$
    – roman
    Jan 25 at 15:42










  • $begingroup$
    I mean your question can be solved with PMI but not those sketches
    $endgroup$
    – Abhas Kumar Sinha
    Jan 25 at 15:46
















11












$begingroup$



Show that:
$$
forall n inBbb N: e < left(1+{1over n}right)^n left(1 + {1over 2n}right)
$$




Till now i've only worked out a couple of proof sketches, and I don't have an idea how to proceed with them.



Please note that this question has already been asked here. The answer there uses derivatives and integrals which i'm not allowed to use.



First sketch



Consider the sequence:
$$
x_n = left(1+{1over n}right)^n left(1 + {1over 2n}right)
$$



One of the ways to show what's required is to show that:
$$
x_{n+1} le x_n
$$

namely the sequence is monotonically decreasing. Now given $ninBbb N$ we may calculate $x_1$:
$$
x_1 = left(1+{1over 1}right)^1left(1+{1over 2cdot 1}right) = 3
$$



Consider the limit:
$$
lim_{ntoinfty}x_n = lim_{ntoinfty} left(1+{1over n}right)^n left(1 + {1over 2n}right) = e
$$



Now based on the fact that the sequence tends to $e$ and it is monotonically decreasing and $x_1 = 3$, then it should follow that:
$$
forall ninBbb N: x_n ge e
$$

Here comes the hard part, I could't prove that $x_n$ is monotonically decreasing. I've considered the fraction:
$$
{x_{n+1}over x_n} = left(1 - {1over (n+1)^2}right)^n cdot frac{2n(2n+3)(n+2)}{(2n+2)(2n+1)(n+1)}
$$



Not sure how to show it is less than $1$.



Second sketch



This sketch is based on the idea from my previous question. Namely it has been shown there that:
$$
e le frac{n+2}{(n+1)(n+1)!} + sum_{k=0}^n {1over k!}
$$



It looks like:
$$
e le frac{n+2}{(n+1)(n+1)!} + sum_{k=0}^n {1over k!} le left(1+{1over n}right)^n left(1 + {1over 2n}right) tag1
$$



In such case if we prove $(1)$ we are done. I've given it several tries but it gets ugly very soon.



The question is:




Is it possible to utilize any of those sketches to finish the proof? If not what would be the way to prove the inequality using anything before the definition of a derivative/Taylor series/intergrals




Thank you!










share|cite|improve this question









$endgroup$












  • $begingroup$
    why not use the Principle of Mathematical Induction? It's just 4 lines if you use that :P XD :)
    $endgroup$
    – Abhas Kumar Sinha
    Jan 25 at 15:27










  • $begingroup$
    @AbhasKumarSinha I would appreciate if you could share that idea
    $endgroup$
    – roman
    Jan 25 at 15:42










  • $begingroup$
    I mean your question can be solved with PMI but not those sketches
    $endgroup$
    – Abhas Kumar Sinha
    Jan 25 at 15:46














11












11








11


1



$begingroup$



Show that:
$$
forall n inBbb N: e < left(1+{1over n}right)^n left(1 + {1over 2n}right)
$$




Till now i've only worked out a couple of proof sketches, and I don't have an idea how to proceed with them.



Please note that this question has already been asked here. The answer there uses derivatives and integrals which i'm not allowed to use.



First sketch



Consider the sequence:
$$
x_n = left(1+{1over n}right)^n left(1 + {1over 2n}right)
$$



One of the ways to show what's required is to show that:
$$
x_{n+1} le x_n
$$

namely the sequence is monotonically decreasing. Now given $ninBbb N$ we may calculate $x_1$:
$$
x_1 = left(1+{1over 1}right)^1left(1+{1over 2cdot 1}right) = 3
$$



Consider the limit:
$$
lim_{ntoinfty}x_n = lim_{ntoinfty} left(1+{1over n}right)^n left(1 + {1over 2n}right) = e
$$



Now based on the fact that the sequence tends to $e$ and it is monotonically decreasing and $x_1 = 3$, then it should follow that:
$$
forall ninBbb N: x_n ge e
$$

Here comes the hard part, I could't prove that $x_n$ is monotonically decreasing. I've considered the fraction:
$$
{x_{n+1}over x_n} = left(1 - {1over (n+1)^2}right)^n cdot frac{2n(2n+3)(n+2)}{(2n+2)(2n+1)(n+1)}
$$



Not sure how to show it is less than $1$.



Second sketch



This sketch is based on the idea from my previous question. Namely it has been shown there that:
$$
e le frac{n+2}{(n+1)(n+1)!} + sum_{k=0}^n {1over k!}
$$



It looks like:
$$
e le frac{n+2}{(n+1)(n+1)!} + sum_{k=0}^n {1over k!} le left(1+{1over n}right)^n left(1 + {1over 2n}right) tag1
$$



In such case if we prove $(1)$ we are done. I've given it several tries but it gets ugly very soon.



The question is:




Is it possible to utilize any of those sketches to finish the proof? If not what would be the way to prove the inequality using anything before the definition of a derivative/Taylor series/intergrals




Thank you!










share|cite|improve this question









$endgroup$





Show that:
$$
forall n inBbb N: e < left(1+{1over n}right)^n left(1 + {1over 2n}right)
$$




Till now i've only worked out a couple of proof sketches, and I don't have an idea how to proceed with them.



Please note that this question has already been asked here. The answer there uses derivatives and integrals which i'm not allowed to use.



First sketch



Consider the sequence:
$$
x_n = left(1+{1over n}right)^n left(1 + {1over 2n}right)
$$



One of the ways to show what's required is to show that:
$$
x_{n+1} le x_n
$$

namely the sequence is monotonically decreasing. Now given $ninBbb N$ we may calculate $x_1$:
$$
x_1 = left(1+{1over 1}right)^1left(1+{1over 2cdot 1}right) = 3
$$



Consider the limit:
$$
lim_{ntoinfty}x_n = lim_{ntoinfty} left(1+{1over n}right)^n left(1 + {1over 2n}right) = e
$$



Now based on the fact that the sequence tends to $e$ and it is monotonically decreasing and $x_1 = 3$, then it should follow that:
$$
forall ninBbb N: x_n ge e
$$

Here comes the hard part, I could't prove that $x_n$ is monotonically decreasing. I've considered the fraction:
$$
{x_{n+1}over x_n} = left(1 - {1over (n+1)^2}right)^n cdot frac{2n(2n+3)(n+2)}{(2n+2)(2n+1)(n+1)}
$$



Not sure how to show it is less than $1$.



Second sketch



This sketch is based on the idea from my previous question. Namely it has been shown there that:
$$
e le frac{n+2}{(n+1)(n+1)!} + sum_{k=0}^n {1over k!}
$$



It looks like:
$$
e le frac{n+2}{(n+1)(n+1)!} + sum_{k=0}^n {1over k!} le left(1+{1over n}right)^n left(1 + {1over 2n}right) tag1
$$



In such case if we prove $(1)$ we are done. I've given it several tries but it gets ugly very soon.



The question is:




Is it possible to utilize any of those sketches to finish the proof? If not what would be the way to prove the inequality using anything before the definition of a derivative/Taylor series/intergrals




Thank you!







calculus sequences-and-series limits proof-verification inequality






share|cite|improve this question













share|cite|improve this question











share|cite|improve this question




share|cite|improve this question










asked Jan 25 at 11:45









romanroman

2,34321224




2,34321224












  • $begingroup$
    why not use the Principle of Mathematical Induction? It's just 4 lines if you use that :P XD :)
    $endgroup$
    – Abhas Kumar Sinha
    Jan 25 at 15:27










  • $begingroup$
    @AbhasKumarSinha I would appreciate if you could share that idea
    $endgroup$
    – roman
    Jan 25 at 15:42










  • $begingroup$
    I mean your question can be solved with PMI but not those sketches
    $endgroup$
    – Abhas Kumar Sinha
    Jan 25 at 15:46


















  • $begingroup$
    why not use the Principle of Mathematical Induction? It's just 4 lines if you use that :P XD :)
    $endgroup$
    – Abhas Kumar Sinha
    Jan 25 at 15:27










  • $begingroup$
    @AbhasKumarSinha I would appreciate if you could share that idea
    $endgroup$
    – roman
    Jan 25 at 15:42










  • $begingroup$
    I mean your question can be solved with PMI but not those sketches
    $endgroup$
    – Abhas Kumar Sinha
    Jan 25 at 15:46
















$begingroup$
why not use the Principle of Mathematical Induction? It's just 4 lines if you use that :P XD :)
$endgroup$
– Abhas Kumar Sinha
Jan 25 at 15:27




$begingroup$
why not use the Principle of Mathematical Induction? It's just 4 lines if you use that :P XD :)
$endgroup$
– Abhas Kumar Sinha
Jan 25 at 15:27












$begingroup$
@AbhasKumarSinha I would appreciate if you could share that idea
$endgroup$
– roman
Jan 25 at 15:42




$begingroup$
@AbhasKumarSinha I would appreciate if you could share that idea
$endgroup$
– roman
Jan 25 at 15:42












$begingroup$
I mean your question can be solved with PMI but not those sketches
$endgroup$
– Abhas Kumar Sinha
Jan 25 at 15:46




$begingroup$
I mean your question can be solved with PMI but not those sketches
$endgroup$
– Abhas Kumar Sinha
Jan 25 at 15:46










3 Answers
3






active

oldest

votes


















2












$begingroup$

The first sketch works.



We need to prove that $$frac{x_n}{x_{n+1}}>1$$ or
$$frac{left(1+frac{1}{n}right)^nleft(1+frac{1}{2n}right)}{left(1+frac{1}{n+1}right)^{n+1}left(1+frac{1}{2n+2}right)}>1$$ or
$$frac{(n+1)^n(2n+1)(n+1)^{n+1}(2n+2)}{n^n(n+2)^{n+1}2n(2n+3)}>1$$ or
$$frac{(n+1)^{2n+2}}{n^{n+1}(n+2)^{n+1}}>frac{2n+3}{2n+1}$$ or$$left(frac{n^2+2n+1}{n^2+2n}right)^{n+1}>frac{2n+3}{2n+1},$$
which is true because
$$left(frac{n^2+2n+1}{n^2+2n}right)^{n+1}=left(1+frac{1}{n^2+2n}right)^{n+1}>1+frac{n+1}{n^2+2n}+frac{(n+1)n}{2(n^2+2n)^2}>frac{2n+3}{2n+1}.$$






share|cite|improve this answer











$endgroup$













  • $begingroup$
    Thank you for the answer, could you please elaborate on where the first inequality comes from? As for the second part you've used binomial expansion, right?
    $endgroup$
    – roman
    Jan 25 at 12:52








  • 1




    $begingroup$
    @roman I added something. See now. Yes, I used the binomial expansion.
    $endgroup$
    – Michael Rozenberg
    Jan 25 at 13:11












  • $begingroup$
    That's a lot of details, thank you so much! And i've been able to follow with the last part by binomial expansion.
    $endgroup$
    – roman
    Jan 25 at 13:38








  • 1




    $begingroup$
    +1, but I found the last step a slog to verify - did I miss a shortcut?
    $endgroup$
    – Calum Gilhooley
    Jan 25 at 19:52






  • 1




    $begingroup$
    @CalumGilhooley it also took me some time, had to expand to a polynomial of $4$-th degree.
    $endgroup$
    – roman
    Jan 25 at 21:23



















2












$begingroup$

$$frac{log(1+x)}{x}+logleft(1+frac{x}{2}right)=1+frac{5x^2}{24}-frac{5x^3}{24}+frac{59 x^4}{320}-ldots=1+sum_{mgeq 2}a_m x^m $$
for any $xin(0,1)$, where the sequence ${a_m}_{mgeq 2}$ can be proved to have alternating signs and the sequence ${|a_m|}_{mgeq 2}$ can be checked to be weakly decreasing by induction on $m$. It follows that the LHS is $>1$ for any $xin(0,1)$: your inequality follows by exponentiating both sides (with $x$ being chosen as $frac{1}{n}$) and by checking $n=1$ as a separate case.






share|cite|improve this answer









$endgroup$





















    1












    $begingroup$

    Another short proof exists, if you use $~displaystyleleft(1+frac{1}{n}right)^{frac{1}{2}}<1+frac{1}{2n}~$ and $~displaystyle e<left(1+frac{1}{n}right)^{n+frac{1}{2}}$



    which is proofed in the note of my question here .






    share|cite|improve this answer









    $endgroup$













    • $begingroup$
      Hm, nice approach. Thanks for sharing this.
      $endgroup$
      – roman
      Feb 1 at 12:56








    • 1




      $begingroup$
      @Roman : You are welcome. It was a pleasure. :)
      $endgroup$
      – user90369
      Feb 1 at 13:15












    • $begingroup$
      I was thinking of mentioning this approach in a comment on Jack D'Aurizio's answer, because he gave a very nice proof of the monotonicity of $(1+frac{1}{n})^{n+frac{1}{2}}$ here.
      $endgroup$
      – Calum Gilhooley
      Feb 1 at 14:27










    • $begingroup$
      @CalumGilhooley : Even if monotony isn't absolutely necessary here, it's of course always good and interesting to see different methods of proof. And so it's good to mentioned it. :)
      $endgroup$
      – user90369
      Feb 1 at 15:29













    Your Answer





    StackExchange.ifUsing("editor", function () {
    return StackExchange.using("mathjaxEditing", function () {
    StackExchange.MarkdownEditor.creationCallbacks.add(function (editor, postfix) {
    StackExchange.mathjaxEditing.prepareWmdForMathJax(editor, postfix, [["$", "$"], ["\\(","\\)"]]);
    });
    });
    }, "mathjax-editing");

    StackExchange.ready(function() {
    var channelOptions = {
    tags: "".split(" "),
    id: "69"
    };
    initTagRenderer("".split(" "), "".split(" "), channelOptions);

    StackExchange.using("externalEditor", function() {
    // Have to fire editor after snippets, if snippets enabled
    if (StackExchange.settings.snippets.snippetsEnabled) {
    StackExchange.using("snippets", function() {
    createEditor();
    });
    }
    else {
    createEditor();
    }
    });

    function createEditor() {
    StackExchange.prepareEditor({
    heartbeatType: 'answer',
    autoActivateHeartbeat: false,
    convertImagesToLinks: true,
    noModals: true,
    showLowRepImageUploadWarning: true,
    reputationToPostImages: 10,
    bindNavPrevention: true,
    postfix: "",
    imageUploader: {
    brandingHtml: "Powered by u003ca class="icon-imgur-white" href="https://imgur.com/"u003eu003c/au003e",
    contentPolicyHtml: "User contributions licensed under u003ca href="https://creativecommons.org/licenses/by-sa/3.0/"u003ecc by-sa 3.0 with attribution requiredu003c/au003e u003ca href="https://stackoverflow.com/legal/content-policy"u003e(content policy)u003c/au003e",
    allowUrls: true
    },
    noCode: true, onDemand: true,
    discardSelector: ".discard-answer"
    ,immediatelyShowMarkdownHelp:true
    });


    }
    });














    draft saved

    draft discarded


















    StackExchange.ready(
    function () {
    StackExchange.openid.initPostLogin('.new-post-login', 'https%3a%2f%2fmath.stackexchange.com%2fquestions%2f3087004%2fshow-that-forall-n-in-bbb-n-e-left11-over-n-rightn-left1-1-ove%23new-answer', 'question_page');
    }
    );

    Post as a guest















    Required, but never shown

























    3 Answers
    3






    active

    oldest

    votes








    3 Answers
    3






    active

    oldest

    votes









    active

    oldest

    votes






    active

    oldest

    votes









    2












    $begingroup$

    The first sketch works.



    We need to prove that $$frac{x_n}{x_{n+1}}>1$$ or
    $$frac{left(1+frac{1}{n}right)^nleft(1+frac{1}{2n}right)}{left(1+frac{1}{n+1}right)^{n+1}left(1+frac{1}{2n+2}right)}>1$$ or
    $$frac{(n+1)^n(2n+1)(n+1)^{n+1}(2n+2)}{n^n(n+2)^{n+1}2n(2n+3)}>1$$ or
    $$frac{(n+1)^{2n+2}}{n^{n+1}(n+2)^{n+1}}>frac{2n+3}{2n+1}$$ or$$left(frac{n^2+2n+1}{n^2+2n}right)^{n+1}>frac{2n+3}{2n+1},$$
    which is true because
    $$left(frac{n^2+2n+1}{n^2+2n}right)^{n+1}=left(1+frac{1}{n^2+2n}right)^{n+1}>1+frac{n+1}{n^2+2n}+frac{(n+1)n}{2(n^2+2n)^2}>frac{2n+3}{2n+1}.$$






    share|cite|improve this answer











    $endgroup$













    • $begingroup$
      Thank you for the answer, could you please elaborate on where the first inequality comes from? As for the second part you've used binomial expansion, right?
      $endgroup$
      – roman
      Jan 25 at 12:52








    • 1




      $begingroup$
      @roman I added something. See now. Yes, I used the binomial expansion.
      $endgroup$
      – Michael Rozenberg
      Jan 25 at 13:11












    • $begingroup$
      That's a lot of details, thank you so much! And i've been able to follow with the last part by binomial expansion.
      $endgroup$
      – roman
      Jan 25 at 13:38








    • 1




      $begingroup$
      +1, but I found the last step a slog to verify - did I miss a shortcut?
      $endgroup$
      – Calum Gilhooley
      Jan 25 at 19:52






    • 1




      $begingroup$
      @CalumGilhooley it also took me some time, had to expand to a polynomial of $4$-th degree.
      $endgroup$
      – roman
      Jan 25 at 21:23
















    2












    $begingroup$

    The first sketch works.



    We need to prove that $$frac{x_n}{x_{n+1}}>1$$ or
    $$frac{left(1+frac{1}{n}right)^nleft(1+frac{1}{2n}right)}{left(1+frac{1}{n+1}right)^{n+1}left(1+frac{1}{2n+2}right)}>1$$ or
    $$frac{(n+1)^n(2n+1)(n+1)^{n+1}(2n+2)}{n^n(n+2)^{n+1}2n(2n+3)}>1$$ or
    $$frac{(n+1)^{2n+2}}{n^{n+1}(n+2)^{n+1}}>frac{2n+3}{2n+1}$$ or$$left(frac{n^2+2n+1}{n^2+2n}right)^{n+1}>frac{2n+3}{2n+1},$$
    which is true because
    $$left(frac{n^2+2n+1}{n^2+2n}right)^{n+1}=left(1+frac{1}{n^2+2n}right)^{n+1}>1+frac{n+1}{n^2+2n}+frac{(n+1)n}{2(n^2+2n)^2}>frac{2n+3}{2n+1}.$$






    share|cite|improve this answer











    $endgroup$













    • $begingroup$
      Thank you for the answer, could you please elaborate on where the first inequality comes from? As for the second part you've used binomial expansion, right?
      $endgroup$
      – roman
      Jan 25 at 12:52








    • 1




      $begingroup$
      @roman I added something. See now. Yes, I used the binomial expansion.
      $endgroup$
      – Michael Rozenberg
      Jan 25 at 13:11












    • $begingroup$
      That's a lot of details, thank you so much! And i've been able to follow with the last part by binomial expansion.
      $endgroup$
      – roman
      Jan 25 at 13:38








    • 1




      $begingroup$
      +1, but I found the last step a slog to verify - did I miss a shortcut?
      $endgroup$
      – Calum Gilhooley
      Jan 25 at 19:52






    • 1




      $begingroup$
      @CalumGilhooley it also took me some time, had to expand to a polynomial of $4$-th degree.
      $endgroup$
      – roman
      Jan 25 at 21:23














    2












    2








    2





    $begingroup$

    The first sketch works.



    We need to prove that $$frac{x_n}{x_{n+1}}>1$$ or
    $$frac{left(1+frac{1}{n}right)^nleft(1+frac{1}{2n}right)}{left(1+frac{1}{n+1}right)^{n+1}left(1+frac{1}{2n+2}right)}>1$$ or
    $$frac{(n+1)^n(2n+1)(n+1)^{n+1}(2n+2)}{n^n(n+2)^{n+1}2n(2n+3)}>1$$ or
    $$frac{(n+1)^{2n+2}}{n^{n+1}(n+2)^{n+1}}>frac{2n+3}{2n+1}$$ or$$left(frac{n^2+2n+1}{n^2+2n}right)^{n+1}>frac{2n+3}{2n+1},$$
    which is true because
    $$left(frac{n^2+2n+1}{n^2+2n}right)^{n+1}=left(1+frac{1}{n^2+2n}right)^{n+1}>1+frac{n+1}{n^2+2n}+frac{(n+1)n}{2(n^2+2n)^2}>frac{2n+3}{2n+1}.$$






    share|cite|improve this answer











    $endgroup$



    The first sketch works.



    We need to prove that $$frac{x_n}{x_{n+1}}>1$$ or
    $$frac{left(1+frac{1}{n}right)^nleft(1+frac{1}{2n}right)}{left(1+frac{1}{n+1}right)^{n+1}left(1+frac{1}{2n+2}right)}>1$$ or
    $$frac{(n+1)^n(2n+1)(n+1)^{n+1}(2n+2)}{n^n(n+2)^{n+1}2n(2n+3)}>1$$ or
    $$frac{(n+1)^{2n+2}}{n^{n+1}(n+2)^{n+1}}>frac{2n+3}{2n+1}$$ or$$left(frac{n^2+2n+1}{n^2+2n}right)^{n+1}>frac{2n+3}{2n+1},$$
    which is true because
    $$left(frac{n^2+2n+1}{n^2+2n}right)^{n+1}=left(1+frac{1}{n^2+2n}right)^{n+1}>1+frac{n+1}{n^2+2n}+frac{(n+1)n}{2(n^2+2n)^2}>frac{2n+3}{2n+1}.$$







    share|cite|improve this answer














    share|cite|improve this answer



    share|cite|improve this answer








    edited Jan 25 at 13:10

























    answered Jan 25 at 12:41









    Michael RozenbergMichael Rozenberg

    107k1895199




    107k1895199












    • $begingroup$
      Thank you for the answer, could you please elaborate on where the first inequality comes from? As for the second part you've used binomial expansion, right?
      $endgroup$
      – roman
      Jan 25 at 12:52








    • 1




      $begingroup$
      @roman I added something. See now. Yes, I used the binomial expansion.
      $endgroup$
      – Michael Rozenberg
      Jan 25 at 13:11












    • $begingroup$
      That's a lot of details, thank you so much! And i've been able to follow with the last part by binomial expansion.
      $endgroup$
      – roman
      Jan 25 at 13:38








    • 1




      $begingroup$
      +1, but I found the last step a slog to verify - did I miss a shortcut?
      $endgroup$
      – Calum Gilhooley
      Jan 25 at 19:52






    • 1




      $begingroup$
      @CalumGilhooley it also took me some time, had to expand to a polynomial of $4$-th degree.
      $endgroup$
      – roman
      Jan 25 at 21:23


















    • $begingroup$
      Thank you for the answer, could you please elaborate on where the first inequality comes from? As for the second part you've used binomial expansion, right?
      $endgroup$
      – roman
      Jan 25 at 12:52








    • 1




      $begingroup$
      @roman I added something. See now. Yes, I used the binomial expansion.
      $endgroup$
      – Michael Rozenberg
      Jan 25 at 13:11












    • $begingroup$
      That's a lot of details, thank you so much! And i've been able to follow with the last part by binomial expansion.
      $endgroup$
      – roman
      Jan 25 at 13:38








    • 1




      $begingroup$
      +1, but I found the last step a slog to verify - did I miss a shortcut?
      $endgroup$
      – Calum Gilhooley
      Jan 25 at 19:52






    • 1




      $begingroup$
      @CalumGilhooley it also took me some time, had to expand to a polynomial of $4$-th degree.
      $endgroup$
      – roman
      Jan 25 at 21:23
















    $begingroup$
    Thank you for the answer, could you please elaborate on where the first inequality comes from? As for the second part you've used binomial expansion, right?
    $endgroup$
    – roman
    Jan 25 at 12:52






    $begingroup$
    Thank you for the answer, could you please elaborate on where the first inequality comes from? As for the second part you've used binomial expansion, right?
    $endgroup$
    – roman
    Jan 25 at 12:52






    1




    1




    $begingroup$
    @roman I added something. See now. Yes, I used the binomial expansion.
    $endgroup$
    – Michael Rozenberg
    Jan 25 at 13:11






    $begingroup$
    @roman I added something. See now. Yes, I used the binomial expansion.
    $endgroup$
    – Michael Rozenberg
    Jan 25 at 13:11














    $begingroup$
    That's a lot of details, thank you so much! And i've been able to follow with the last part by binomial expansion.
    $endgroup$
    – roman
    Jan 25 at 13:38






    $begingroup$
    That's a lot of details, thank you so much! And i've been able to follow with the last part by binomial expansion.
    $endgroup$
    – roman
    Jan 25 at 13:38






    1




    1




    $begingroup$
    +1, but I found the last step a slog to verify - did I miss a shortcut?
    $endgroup$
    – Calum Gilhooley
    Jan 25 at 19:52




    $begingroup$
    +1, but I found the last step a slog to verify - did I miss a shortcut?
    $endgroup$
    – Calum Gilhooley
    Jan 25 at 19:52




    1




    1




    $begingroup$
    @CalumGilhooley it also took me some time, had to expand to a polynomial of $4$-th degree.
    $endgroup$
    – roman
    Jan 25 at 21:23




    $begingroup$
    @CalumGilhooley it also took me some time, had to expand to a polynomial of $4$-th degree.
    $endgroup$
    – roman
    Jan 25 at 21:23











    2












    $begingroup$

    $$frac{log(1+x)}{x}+logleft(1+frac{x}{2}right)=1+frac{5x^2}{24}-frac{5x^3}{24}+frac{59 x^4}{320}-ldots=1+sum_{mgeq 2}a_m x^m $$
    for any $xin(0,1)$, where the sequence ${a_m}_{mgeq 2}$ can be proved to have alternating signs and the sequence ${|a_m|}_{mgeq 2}$ can be checked to be weakly decreasing by induction on $m$. It follows that the LHS is $>1$ for any $xin(0,1)$: your inequality follows by exponentiating both sides (with $x$ being chosen as $frac{1}{n}$) and by checking $n=1$ as a separate case.






    share|cite|improve this answer









    $endgroup$


















      2












      $begingroup$

      $$frac{log(1+x)}{x}+logleft(1+frac{x}{2}right)=1+frac{5x^2}{24}-frac{5x^3}{24}+frac{59 x^4}{320}-ldots=1+sum_{mgeq 2}a_m x^m $$
      for any $xin(0,1)$, where the sequence ${a_m}_{mgeq 2}$ can be proved to have alternating signs and the sequence ${|a_m|}_{mgeq 2}$ can be checked to be weakly decreasing by induction on $m$. It follows that the LHS is $>1$ for any $xin(0,1)$: your inequality follows by exponentiating both sides (with $x$ being chosen as $frac{1}{n}$) and by checking $n=1$ as a separate case.






      share|cite|improve this answer









      $endgroup$
















        2












        2








        2





        $begingroup$

        $$frac{log(1+x)}{x}+logleft(1+frac{x}{2}right)=1+frac{5x^2}{24}-frac{5x^3}{24}+frac{59 x^4}{320}-ldots=1+sum_{mgeq 2}a_m x^m $$
        for any $xin(0,1)$, where the sequence ${a_m}_{mgeq 2}$ can be proved to have alternating signs and the sequence ${|a_m|}_{mgeq 2}$ can be checked to be weakly decreasing by induction on $m$. It follows that the LHS is $>1$ for any $xin(0,1)$: your inequality follows by exponentiating both sides (with $x$ being chosen as $frac{1}{n}$) and by checking $n=1$ as a separate case.






        share|cite|improve this answer









        $endgroup$



        $$frac{log(1+x)}{x}+logleft(1+frac{x}{2}right)=1+frac{5x^2}{24}-frac{5x^3}{24}+frac{59 x^4}{320}-ldots=1+sum_{mgeq 2}a_m x^m $$
        for any $xin(0,1)$, where the sequence ${a_m}_{mgeq 2}$ can be proved to have alternating signs and the sequence ${|a_m|}_{mgeq 2}$ can be checked to be weakly decreasing by induction on $m$. It follows that the LHS is $>1$ for any $xin(0,1)$: your inequality follows by exponentiating both sides (with $x$ being chosen as $frac{1}{n}$) and by checking $n=1$ as a separate case.







        share|cite|improve this answer












        share|cite|improve this answer



        share|cite|improve this answer










        answered Jan 25 at 17:44









        Jack D'AurizioJack D'Aurizio

        291k33284667




        291k33284667























            1












            $begingroup$

            Another short proof exists, if you use $~displaystyleleft(1+frac{1}{n}right)^{frac{1}{2}}<1+frac{1}{2n}~$ and $~displaystyle e<left(1+frac{1}{n}right)^{n+frac{1}{2}}$



            which is proofed in the note of my question here .






            share|cite|improve this answer









            $endgroup$













            • $begingroup$
              Hm, nice approach. Thanks for sharing this.
              $endgroup$
              – roman
              Feb 1 at 12:56








            • 1




              $begingroup$
              @Roman : You are welcome. It was a pleasure. :)
              $endgroup$
              – user90369
              Feb 1 at 13:15












            • $begingroup$
              I was thinking of mentioning this approach in a comment on Jack D'Aurizio's answer, because he gave a very nice proof of the monotonicity of $(1+frac{1}{n})^{n+frac{1}{2}}$ here.
              $endgroup$
              – Calum Gilhooley
              Feb 1 at 14:27










            • $begingroup$
              @CalumGilhooley : Even if monotony isn't absolutely necessary here, it's of course always good and interesting to see different methods of proof. And so it's good to mentioned it. :)
              $endgroup$
              – user90369
              Feb 1 at 15:29


















            1












            $begingroup$

            Another short proof exists, if you use $~displaystyleleft(1+frac{1}{n}right)^{frac{1}{2}}<1+frac{1}{2n}~$ and $~displaystyle e<left(1+frac{1}{n}right)^{n+frac{1}{2}}$



            which is proofed in the note of my question here .






            share|cite|improve this answer









            $endgroup$













            • $begingroup$
              Hm, nice approach. Thanks for sharing this.
              $endgroup$
              – roman
              Feb 1 at 12:56








            • 1




              $begingroup$
              @Roman : You are welcome. It was a pleasure. :)
              $endgroup$
              – user90369
              Feb 1 at 13:15












            • $begingroup$
              I was thinking of mentioning this approach in a comment on Jack D'Aurizio's answer, because he gave a very nice proof of the monotonicity of $(1+frac{1}{n})^{n+frac{1}{2}}$ here.
              $endgroup$
              – Calum Gilhooley
              Feb 1 at 14:27










            • $begingroup$
              @CalumGilhooley : Even if monotony isn't absolutely necessary here, it's of course always good and interesting to see different methods of proof. And so it's good to mentioned it. :)
              $endgroup$
              – user90369
              Feb 1 at 15:29
















            1












            1








            1





            $begingroup$

            Another short proof exists, if you use $~displaystyleleft(1+frac{1}{n}right)^{frac{1}{2}}<1+frac{1}{2n}~$ and $~displaystyle e<left(1+frac{1}{n}right)^{n+frac{1}{2}}$



            which is proofed in the note of my question here .






            share|cite|improve this answer









            $endgroup$



            Another short proof exists, if you use $~displaystyleleft(1+frac{1}{n}right)^{frac{1}{2}}<1+frac{1}{2n}~$ and $~displaystyle e<left(1+frac{1}{n}right)^{n+frac{1}{2}}$



            which is proofed in the note of my question here .







            share|cite|improve this answer












            share|cite|improve this answer



            share|cite|improve this answer










            answered Feb 1 at 12:31









            user90369user90369

            8,395925




            8,395925












            • $begingroup$
              Hm, nice approach. Thanks for sharing this.
              $endgroup$
              – roman
              Feb 1 at 12:56








            • 1




              $begingroup$
              @Roman : You are welcome. It was a pleasure. :)
              $endgroup$
              – user90369
              Feb 1 at 13:15












            • $begingroup$
              I was thinking of mentioning this approach in a comment on Jack D'Aurizio's answer, because he gave a very nice proof of the monotonicity of $(1+frac{1}{n})^{n+frac{1}{2}}$ here.
              $endgroup$
              – Calum Gilhooley
              Feb 1 at 14:27










            • $begingroup$
              @CalumGilhooley : Even if monotony isn't absolutely necessary here, it's of course always good and interesting to see different methods of proof. And so it's good to mentioned it. :)
              $endgroup$
              – user90369
              Feb 1 at 15:29




















            • $begingroup$
              Hm, nice approach. Thanks for sharing this.
              $endgroup$
              – roman
              Feb 1 at 12:56








            • 1




              $begingroup$
              @Roman : You are welcome. It was a pleasure. :)
              $endgroup$
              – user90369
              Feb 1 at 13:15












            • $begingroup$
              I was thinking of mentioning this approach in a comment on Jack D'Aurizio's answer, because he gave a very nice proof of the monotonicity of $(1+frac{1}{n})^{n+frac{1}{2}}$ here.
              $endgroup$
              – Calum Gilhooley
              Feb 1 at 14:27










            • $begingroup$
              @CalumGilhooley : Even if monotony isn't absolutely necessary here, it's of course always good and interesting to see different methods of proof. And so it's good to mentioned it. :)
              $endgroup$
              – user90369
              Feb 1 at 15:29


















            $begingroup$
            Hm, nice approach. Thanks for sharing this.
            $endgroup$
            – roman
            Feb 1 at 12:56






            $begingroup$
            Hm, nice approach. Thanks for sharing this.
            $endgroup$
            – roman
            Feb 1 at 12:56






            1




            1




            $begingroup$
            @Roman : You are welcome. It was a pleasure. :)
            $endgroup$
            – user90369
            Feb 1 at 13:15






            $begingroup$
            @Roman : You are welcome. It was a pleasure. :)
            $endgroup$
            – user90369
            Feb 1 at 13:15














            $begingroup$
            I was thinking of mentioning this approach in a comment on Jack D'Aurizio's answer, because he gave a very nice proof of the monotonicity of $(1+frac{1}{n})^{n+frac{1}{2}}$ here.
            $endgroup$
            – Calum Gilhooley
            Feb 1 at 14:27




            $begingroup$
            I was thinking of mentioning this approach in a comment on Jack D'Aurizio's answer, because he gave a very nice proof of the monotonicity of $(1+frac{1}{n})^{n+frac{1}{2}}$ here.
            $endgroup$
            – Calum Gilhooley
            Feb 1 at 14:27












            $begingroup$
            @CalumGilhooley : Even if monotony isn't absolutely necessary here, it's of course always good and interesting to see different methods of proof. And so it's good to mentioned it. :)
            $endgroup$
            – user90369
            Feb 1 at 15:29






            $begingroup$
            @CalumGilhooley : Even if monotony isn't absolutely necessary here, it's of course always good and interesting to see different methods of proof. And so it's good to mentioned it. :)
            $endgroup$
            – user90369
            Feb 1 at 15:29




















            draft saved

            draft discarded




















































            Thanks for contributing an answer to Mathematics Stack Exchange!


            • Please be sure to answer the question. Provide details and share your research!

            But avoid



            • Asking for help, clarification, or responding to other answers.

            • Making statements based on opinion; back them up with references or personal experience.


            Use MathJax to format equations. MathJax reference.


            To learn more, see our tips on writing great answers.




            draft saved


            draft discarded














            StackExchange.ready(
            function () {
            StackExchange.openid.initPostLogin('.new-post-login', 'https%3a%2f%2fmath.stackexchange.com%2fquestions%2f3087004%2fshow-that-forall-n-in-bbb-n-e-left11-over-n-rightn-left1-1-ove%23new-answer', 'question_page');
            }
            );

            Post as a guest















            Required, but never shown





















































            Required, but never shown














            Required, but never shown












            Required, but never shown







            Required, but never shown

































            Required, but never shown














            Required, but never shown












            Required, but never shown







            Required, but never shown







            Popular posts from this blog

            Mario Kart Wii

            What does “Dominus providebit” mean?

            Antonio Litta Visconti Arese